Umkehrfunktion f−1:f(R)→Rf−1:f(R)→Rf^{-1}:f(\mathbb{R})\to\mathbb{R} einer streng steigenden Funktion f:R →Rf:R→Rf:\mathbb{R}\to\mathbb{R} ist stetig

Ich habe die folgende Aussage bewiesen und möchte wissen, ob mein Beweis richtig ist und/oder/ob/wie er verbessert werden kann.

"Vermuten F : R R ist eine streng steigende Funktion.

Beweisen Sie, dass die Umkehrfunktion F 1 : F ( R ) R ist eine stetige Funktion."

Mein Beweis:

Lassen F : R R eine streng steigende Funktion sein: dann ist sie injektiv und als Funktion F : R F ( R ) es muss surjektiv sein, also hat es eine Umkehrung F 1 : F ( R ) R die auch streng steigend sein muss ( 1 ) .

Nehmen Sie jetzt an, dass F 1 waren an einem Punkt diskontinuierlich j D F ( R ) : dann, da es sich um eine wachsende Funktion handelt, j D muss also beim intervall eine sprungstelle sein ICH j D := ( lim j j D ,   j F ( R ) F 1 ( j ) , lim j j D + ,   j F ( R ) F 1 ( j ) ) = ( sup j < j D ,   j F ( R ) F 1 ( j ) , inf j > j D ,   j F ( R ) F 1 ( j ) ) muss nicht leer sein und wir können ein Element auswählen X ¯ X D = F 1 ( j D ) darin also F ( X ¯ ) = j D , sondern Sein F streng hypothetisch zunehmend ist es auch nicht F ( X ¯ ) > j D oder F ( X ¯ ) < j D , ein Widerspruch.

So, F 1 ( R ) R darf an keiner Stelle diskontinuierlich sein, dh es muss durchgehend eingeschaltet sein F ( R ) .


( 1 ) lassen j 1 , j 2 F ( R ) und nehme an, wlog j 1 < j 2 : Dann F 1 ( j 1 ) = F 1 ( F ( X 1 ) ) = X 1 Und F 1 ( j 2 ) = F 1 ( F ( X 2 ) ) = X 2 und wenn X 1 X 2 Dann F ( X 1 ) = j 1 j 2 = F ( X 2 ) Widerspruch, so muss es sein X 1 = F 1 ( j 1 ) < X 2 = F 1 ( j 2 )

Ist die Funktion f stetig?
@EduardoMaza nein, es ist nicht kontinuierlich
@MartinR danke für dein Interesse an meiner Frage und für diese Links, ich habe einen neuen Beweis für diese Aussage gelernt, indem ich den zweiten gelesen habe. Es scheint mir jedoch, dass mein Beweis etwas anders ist als diese beiden: Würden Sie ihn bitte überprüfen und mir sagen, ob Sie ihn für richtig halten?
Beachten Sie, dass die linksseitige und/oder rechtsseitige Grenze möglicherweise nicht existiert, z. B. wenn j D ist ein isolierter Punkt von F ( R ) . Ebenso die Sets { j < j D j F ( R ) } und/oder { j > j D j F ( R ) } leer sein, so dass ihr Supremum bzw. infimum ist nicht definiert.
@Martin R danke für deinen Kommentar; bezüglich Ihrer ersten Bemerkung, seiend F 1 steigend, sollte das linke und rechte Limit nicht immer vorhanden sein ( math.stackexchange.com/q/4171854 ) (selbst wenn j D ist ein isolierter Punkt, siehe math.stackexchange.com/questions/27429/… )?

Antworten (3)

Betrachten Sie die Funktion F 1 : R R gegeben von

F 1 ( j ) := { j 1 j < 0 j j 0.
Hier verwende ich das Label " F 1 „nur formal, nicht als Hinweis darauf, dass es sich um die Umkehrung einer Funktion handelt F (obwohl es natürlich die Umkehrung einer Funktion ist ).

Deutlich, F 1 ist strikt steigend und ist eine Bijektion aus dem Bereich seiner Umkehrung zu R . Vermietung j D = 0 , wir sehen das

( lim j j D ,   j Dom F 1 F 1 ( j ) , lim j j D + ,   j Dom F 1 F 1 ( j ) ) = ( 1 , 0 )
ist sicherlich nicht leer, enthält aber keine Elemente aus dem Bereich von F 1 --das heißt, kein Element der Domäne der Umkehrung von F 1 .

Das ist der Fehler in Ihrer Argumentation. Nur weil ein Intervall nicht leer ist, heißt das nicht, dass es ein Element im Bereich einer beliebigen, streng steigenden Funktion enthält. Mit anderen Worten, Sie haben die folgende Aussage nicht wirklich begründet.

wir können ein Element auswählen X ¯ X D = F 1 ( j D ) in [dem nichtleeren Intervall] so F ( X ¯ ) = j D

Da alles, worüber Sie geschlussfolgert haben F 1 ist, dass es streng steigend ist und eine Bijektion aus dem Bereich seiner Umkehrung zu ist R , dann ist das durchaus möglich F 1 = G , In diesem Fall fällt Ihr Argument hin.

Hinzugefügt : Ein besserer Ansatz wäre, direkt vorzugehen. Nehmen Sie eine beliebige j 0 F [ R ] , und lass X 0 := F 1 ( j 0 ) . Seit F streng ansteigend ist, dann z X < X 0 (bzw. z X > X 0 ) wir haben F ( X ) < j 0 (bzw. F ( X ) > j 0 ).

Nehmen Sie eine beliebige ε > 0 , lassen j M := F ( X 0 ε ) , und lass j M := F ( X 0 + ε ) , so dass j M , j M F [ R ] Und j M < j 0 < j M .

Vermietung δ = Mindest { j 0 j M , j M j 0 } , wir haben δ > 0 , und für alle j R , Wenn | j j 0 | < δ , Dann j M < j < j M .

Nehmen Sie insbesondere keine j F [ R ] so dass | j j 0 | < δ , und lass X = F 1 ( j ) . Seit F ist streng ansteigend und F ( X 0 ε ) = j M < j = F ( X ) , Dann X 0 ε < X . Ähnlich, X < X 0 + ε , und so | X X 0 | < ε , oder gleichwertig,

| F 1 ( j ) F 1 ( j 0 ) | < ε ,
woher wir das gezeigt haben F 1 ist stetig bei j 0 , wie gewünscht.

Ich werde darüber nachdenken! Wenn ich einen glatten und einfachen Beweis finden kann, füge ich ihn meiner Antwort hinzu.

Forderung B = F ( R ) und lass G : B R Sei F ist umgekehrt. Beachten Sie, dass G nimmt ebenfalls stark zu. Vermuten G ist an einer Stelle unterbrochen j B . Lassen a = lim w B , w j G ( w ) Und β = lim z B , z j + G ( z ) . Seit G ist zunehmend und diskontinuierlich bei j , wir haben a < β .

Nehmen X 0 [ a + β a 3 , β β a 3 ] .

Dann für klein genug δ > 0 , das kommt vor G ( j δ ) < X 0 < G ( j + δ ) . Beachten Sie, dass δ hängt nicht davon ab X 0 .

Bewirbt sich F zu den obigen Ungleichungsausbeuten j δ < F ( X 0 ) < j + δ . Vermietung δ 0 , wir glauben, dass F ( X 0 ) = j . So F im nicht entarteten Intervall tatsächlich konstant ist [ a + β a 3 , β β a 3 ] !!

Hier habe ich angenommen j kann mit Punkten angenähert werden B von beiden Seiten. Aber das gleiche Argument kann verwendet werden, wenn j nur ein linker Grenzpunkt oder rechter Grenzpunkt von ist B , dazu müssen Sie nur verwenden G ( j ) anstelle von β Und a , bzw.

Schöne Argumentation!

Zunächst möchte ich @Cameron Buie für seine Bemühungen danken, auf den Fehler im OP-Beweis hinzuweisen.

Natürlich ist die Aussage falsch. Monotonie impliziert fast überall Kontinuität. Selbst eine streng monotone Funktion ist nicht notwendigerweise stetig. Auch nicht seine Umkehrung, die ebenfalls streng monoton ist.

Hier ein einfaches Gegenbeispiel:

F ( X ) = { X + 2 , X > 0 ; 1 , X = 0 ; X , X < 0 .
Die Funktion F : R ] , 0 [ { 1 } ] 2 , [   ist eine streng steigende Bijektion.

Die streng steigende Inverse F 1 : ] , 0 [ { 1 } ] 2 , [ R

F 1 ( X ) = { X 2 , X > 2 ; 0 , X = 1 ; X , X < 0 .
ist diskontinuierlich

Es scheint mir, dass 2 < 0 Aber F ( 2 ) = 2 > 1 = F ( 0 ) So F ist nicht strikt steigend
Der Tippfehler X korrigiert wird X .
Lassen G = F 1 : Dann G gilt eindeutig für X < 0 und auch lim X 0 = 0 = lim X 0 + G ( X ) (Es ist vage wahr, dass die rechte Grenze existiert und gleich ist 0 ) also ist es auch stetig bei 0 ; es ist auch vage wahr, dass lim X 1 G ( X ) = lim X 1 + G ( X ) = G ( 1 ) = 0 (Such dir irgendeine aus ε > 0 Und δ = 1 2 ); es ist eindeutig kontinuierlich für X > 2 Und lim X 2 G ( X ) = lim X 2 + G ( X ) = G ( 2 ) = 0 So G = F 1 ist tatsächlich stetig.
@lorenz. Nein, weil die Grenzen lim X 0 + G Und lim X 2 G existiert nicht. Lassen Sie mich die Diskontinuität auf andere Weise für Sie beweisen. Was ist das umgekehrte Bild des offenen Intervalls? ] 1 / 2 , 3 / 2 [ ? Es ist der Satz { 1 } ] 5 / 2 , 7 / 2 [ was keine offene Menge ist. Überzeugt ?